Intuitive Erklärung dafür, warum dieselbe Kraft, die weiter von einem Scharnier entfernt angewendet wird, eine größere Winkelbeschleunigung verursacht, als wenn sie näher angewendet wird?

Ein Standardbeispiel für ein Drehmomentproblem ist das Öffnen einer Tür – die gleiche Kraft F, die weit vom Scharnier entfernt aufgebracht wird, verursacht eine größere Winkelbeschleunigung als wenn sie nahe am Scharnier aufgebracht wird.

Ich hatte immer Probleme, eine intuitive Erklärung dafür zu finden - wenn Sie sich in der Nähe des Scharniers befinden und die Tür mit aller Kraft drücken, bewegt sie sich kaum - also wo geht die ganze Kraft hin? Vielleicht am Scharnier?

Lassen Sie uns die Tür mit einer Stange modellieren, die an einem Scharnier befestigt ist. Angenommen, die Länge der Stange ist L mit Masse M , gleichmäßig verteilt. Sein Trägheitsmoment ist ICH = M L 2 / 3 . Sie drücken die Stange mit Kraft F A P P l ich e D auf Distanz D , was ihm eine Winkelbeschleunigung gibt A . Verwenden T = ICH A wir bekommen F A P P l ich e D D = ICH A , oder F A P P l ich e D = ICH A / D = A M L 2 / 3 D .

Nun, wenn Sie jedes Massenelement (Teilchen) der Masse betrachten M im Stab erfährt er einzeln eine Tangentialkraft F T A N = M A T A N = M A R , Wo R ist sein Abstand vom Scharnier, und so ist die über alle Teilchen summierte Gesamttangentialkraft F T A N   T Ö T A l = 0 L ( M / L )   A R   D R = A M / 2 L .

Aber nach Newtons 2. Gesetz gilt F T A N   T Ö T A l = Summe aller äußeren Kräfte in tangentialer Richtung. So F T A N   T Ö T A l = F A P P l ich e D + F S Ö M e   Ö T H e R .

Was ist diese andere Kraft, F S Ö M e   Ö T H e R = A M / 2 L A M L 2 / 3 D , die von d (Ihrem Angriffspunkt der Kraft) abhängt? Woher kommt das?

Danke für deine Antwort Markus! Bitte beachten Sie auch meine andere Frage - physical.stackexchange.com/questions/65703/…

Antworten (1)

Sie haben Ihre eigene Frage bereits beantwortet! Es gibt eine Kraft zwischen dem Scharnier und der Tür. Wenn die Tür nicht am Scharnier befestigt wäre, würde sie zusätzlich zum Drehen wegfliegen.

Der einzige Fehler, den Sie haben, ist ein Fehler in Ihrem Integral. Die Nettokraft auf die Tür ist A M L 2 in deiner Notation. (Hinweis: Es ist wahrscheinlich am besten, es nicht zu verwenden A für Winkel- und Linearbeschleunigung. a ist üblich für die Winkelbeschleunigung.)

Für eine feste aufgebrachte Kraft, wenn Sie eliminieren A aus deinem Ausdruck heraus findest du das für eine Bewerbung F , die Kraft aus dem Scharnier ist F H ich N G e = F ( 3 D 2 L 1 ) . An der Grenze D 0 , du drückst direkt am Scharnier und es bricht deinen Stoß genau ab. Wenn du drückst D = 2 3 L , gibt es keine Kraft auf das Scharnier. Dies ist die Stelle, an der, wenn die Tür frei im Raum schweben würde, das Drücken dort die Tür nach vorne bewegen würde, aber sie in Drehung versetzen würde, so dass die Kante der Tür stationär bleibt. Wenn du drückst D = L , das Scharnier schiebt sich tatsächlich mit. Da die Gegenkraft vom Scharnier mit zunehmender abnimmt D , es wird einfacher, die Tür zu schieben, je weiter Sie nach außen gehen.

Re: "wohin die Macht geht". Da Kraft nicht erhalten bleibt, geht sie nirgendwohin, aber Kraft ist eine Änderungsrate des Impulses, und wir können darüber sprechen, wohin der Impuls geht. Auch hier haben Sie die Frage bereits beantwortet. Es gibt einen Impulsfluss von Ihrer Hand durch die Tür und in das Scharnier. Wenn das Scharnier schwach wäre, könnten Sie es brechen, indem Sie auf die Tür in der Nähe des Scharniers drücken, aber die gleiche Kraft, die weiter außen ausgeübt wird, würde das Scharnier nicht brechen.

Ich wollte hinzufügen, dass Kraft keine konservierte Größe ist, und die Auswirkungen. Aber auch das hattest du abgedeckt.